Mathcenter Forum  

Go Back   Mathcenter Forum > คณิตศาสตร์มัธยมศึกษา > ปัญหาคณิตศาสตร์ ม. ต้น > ข้อสอบในโรงเรียน ม.ต้น
สมัครสมาชิก คู่มือการใช้ รายชื่อสมาชิก ปฏิทิน ข้อความวันนี้

ตั้งหัวข้อใหม่ Reply
 
เครื่องมือของหัวข้อ ค้นหาในหัวข้อนี้
  #16  
Old 20 มกราคม 2007, 20:08
prachya prachya ไม่อยู่ในระบบ
กระบี่ไว
 
วันที่สมัครสมาชิก: 10 กรกฎาคม 2005
ข้อความ: 204
prachya is on a distinguished road
Post

ตอนแรก

ข้อ 19 ผมไม่เจอคำตอบนะครับ ไม่ทราบว่าผมคิดผิดหรือเปล่า?

1~8~15~....~99 --> 15 จำนวน
2~9~16~....~100--> 15 จำนวน
3~10~.......~94 --> 14 จำนวน
4~11~.......~95 --> 14 จำนวน
5~12~.......~96 --> 14 จำนวน
6~13~.......~97 --> 14 จำนวน
7~14~.......~98 --> 14 จำนวน

จำนวนวิธีที่เลือกได้ $ {15 \choose 2}x2 + {14 \choose 2} x5 = 210+455 = 665$

ปล. ข้อ 17 นึกรูปไม่ออกอ่ะครับ ข้อ 10 ท่าทางหนักเอาการ = =" ไว้ลองคิดก่อนครับ
ตอบพร้อมอ้างอิงข้อความนี้
  #17  
Old 20 มกราคม 2007, 20:58
prachya prachya ไม่อยู่ในระบบ
กระบี่ไว
 
วันที่สมัครสมาชิก: 10 กรกฎาคม 2005
ข้อความ: 204
prachya is on a distinguished road
Post

ข้อ 10 ครับ

สมมุติให้ก้อนๆตัวเลขทางซ้าย เป็น b ทางขวาเป็น c

b $ \frac{1}{2} $ c
b-c $ \frac{1}{2} $-c c-c
0 $ \frac{1}{2} $-c 0
จะได้ว่า $ \frac{1}{2} $-c = 0
ดังนั้น b = c = $ \frac{1}{2} $

สมมติกรณีที่ n = 2


แต่ถ้าใครมีวิธี ที่ไม่ต้องแทนค่า ก็ลองโพสกันมานะครับ คิดไม่ออกจริงๆ
ตอบพร้อมอ้างอิงข้อความนี้
  #18  
Old 21 มกราคม 2007, 01:16
nooonuii nooonuii ไม่อยู่ในระบบ
ผู้พิทักษ์กฎทั่วไป
 
วันที่สมัครสมาชิก: 25 พฤษภาคม 2001
ข้อความ: 6,408
nooonuii is on a distinguished road
Post

นี่ข้อสอบของเด็กม.ต้นจริงๆเหรอครับ แต่ละข้อทำผมอึ้งไปหลายนาทีเลยครับ

10. My Solution :
แนวคิด : เวลาเจอโจทย์อสมการ สิ่งแรกที่ควรทำคือเช็คเงื่อนไขของสมการครับว่าจะเกิดขึ้นเมื่อไหร่ สิ่งนี้จะนำไปสู่กระบวนการคิดและการเลือกอสมการสำเร็จรูป (AM-GM, Cauchy-Schwarz, Holder, etc) มาใช้ในการแก้ปัญหาได้อย่างเหมาะสมครับ ส่วนใหญ่อสมการที่มีสมมาตรในตัวแปรหรือตัวแปรวนซ้ำกันอย่างข้อนี้สมการมักจะเกิดเมื่อตัวแปรทุกตัวมีค่าเท่ากันครับ ซึ่งถ้าโจทย์มีตัวเลือกมาให้เราแทบจะไม่ต้องคิดต่อเลย

ลองดูวิธีพิสูจน์ข้อนี้แบบเต็มๆครับ

กำหนดให้
$$\displaystyle{ A = \frac{a_1^2}{a_1+a_2} + \frac{a_2^2}{a_2+a_3} + \cdots + \frac{a_n^2}{a_n+a_1}}$$
$$\displaystyle{ B = \frac{a_2^2}{a_1+a_2} + \frac{a_3^2}{a_2+a_3} + \cdots + \frac{a_1^2}{a_n+a_1}}$$

เราจะได้ว่า (อันนี้คือหัวใจสำคัญของโจทย์ข้อนี้ครับ )

$$\begin{array}{rcl} A - B & = & \displaystyle{\frac{a_1^2-a_2^2}{a_1+a_2} + \frac{a_2^2-a_3^2}{a_2+a_3} + \cdots + \frac{a_n^2-a_1^2}{a_n+a_1}} \\ & = & (a_1-a_2)+(a_2-a_3)+\cdots + (a_n-a_1) \\ & = & 0 \end{array} $$
ดังนั้น $\displaystyle{ A=B=\frac{1}{2} }$

ต่อไปเราจะใช้อสมการต่อไปนี้ $$\displaystyle{\frac{x^2+y^2}{x+y}\geq \frac{x+y}{2}}$$ สำหรับจำนวนจริงบวก $x,y$ ใดๆ ซึ่งพิสูจน์ได้ไม่ยากครับ และสมการจะเกิดขึ้นเมื่อ $x=y$

ดังนั้นจะได้ว่า
$$\begin{array}{rcl} A+B & = & \displaystyle{ \frac{a_1^2+a_2^2}{a_1+a_2} + \frac{a_2^2+a_3^2}{a_2+a_3} + \cdots + \frac{a_n^2+a_1^2}{a_n+a_1} } \\ & \geq & \frac{a_1+a_2}{2} +\cdots+\frac{a_n+a_1}{2} \\ & = & a_1+a_2+\cdots + a_n \end{array} $$
และสมการเกิดเมื่อ $a_1=a_2=\cdots = a_n$

แต่เราทราบว่า $A+B = 1$ และ $a_1+a_2+\cdots + a_n = 1$
ดังนั้น $A+B = a_1+a_2+\cdots + a_n $
เราจึงได้ว่า $\displaystyle{ a_1=a_2=\cdots = a_n=\frac{1}{n} }$
แทนค่าเราจะได้ $\displaystyle{ a_1 + a_2^2 + \cdots + a_n^n = \frac{n^n-1}{n^n(n-1)}}$
__________________
site:mathcenter.net คำค้น
ตอบพร้อมอ้างอิงข้อความนี้
  #19  
Old 21 มกราคม 2007, 01:49
nongtum's Avatar
nongtum nongtum ไม่อยู่ในระบบ
ผู้พิทักษ์กฎทั่วไป
 
วันที่สมัครสมาชิก: 10 เมษายน 2005
ข้อความ: 3,246
nongtum is on a distinguished road
Icon15

แว้บมาทำข้อ 17 ตอนแรกครับ

เนื่ืองจากข้อนี้เราไม่ต้องการค่าเป๊ะๆ และความยาวด้านทั้งสามใกล้เคียงกันและใหญ่พอ ก่อนอื่นเราจะพิจารณากรณีสามเหลี่ยมด้านเท่าดังนี้

เราทราบว่าสามเหลี่ยมด้านเท่าที่แต่ละด้านยาว $a$ รัศมีวงกลมแนบในจะเป็น $r=\frac{\triangle}{S}=\frac{\sqrt3}{6}a$
ทำให้ระยะจากจุดยอดไปหา $MN$ เป็น $\frac{\sqrt3}{3}a$ ดังนั้น $MN=\frac{2\sqrt3}{3}a\tan 30^\circ=\frac{2}{3}a$

แทนค่าในโจทย์เข้าไป เราจะได้ $\frac{4010}{3}=1336\frac23<MN<\frac{4014}{3}=1338$
ก็เลยตอบข้อ ค. ครับ
__________________
คนไทยร่วมใจอย่าใช้ภาษาวิบัติ
ฝึกพิมพ์สัญลักษณ์สักนิด ชีวิต(คนตอบและคนถาม)จะง่ายขึ้นเยอะ (จริงๆนะ)

Stay Hungry. Stay Foolish.

21 มกราคม 2007 02:02 : ข้อความนี้ถูกแก้ไขแล้ว 1 ครั้ง, ครั้งล่าสุดโดยคุณ nongtum
ตอบพร้อมอ้างอิงข้อความนี้
  #20  
Old 21 มกราคม 2007, 03:56
nooonuii nooonuii ไม่อยู่ในระบบ
ผู้พิทักษ์กฎทั่วไป
 
วันที่สมัครสมาชิก: 25 พฤษภาคม 2001
ข้อความ: 6,408
nooonuii is on a distinguished road
Post

มาต่อ Jigsaw ให้คุณ nongtum ครับ

ตอนที่ 2

4. ข้อนี้ใครท่องพยัญชนะไม่ครบคงไม่รอดครับ

สังเกตว่าพยัญชนะตัวที่ 2n จะแปรผันตรงกับพยัญชนะตัวที่ 2n+3 และแปรผกผันกับตัวที่ 2n+1,2n+2
เนื่องจาก บ เป็นพยัญชนะตัวที่ 26 เราจะได้ว่า บ แปรผกผันกับ ฮ

9. $\displaystyle{ \frac{x-1}{x^3-x^2-x+1} = \frac{1}{x^2-1} }$

ดังนั้น $A(x+1) + B(x-1) = C+1$
เทียบสัมประสิทธิ์จะได้
$A+B=0$
$A-B = C+1$

แก้สมการหา $A,B$ ในรูปของ $C$ จะได้
$\displaystyle{ A = \frac{C+1}{2}, B = - \frac{C+1}{2}}$
แก้อสมการ $A\geq B\geq C$ จะได้ $C = -1$

10. ขอนี้โหดร้ายสำหรับเด็กม.ต้น มากๆเลยครับ
ให้ $\displaystyle{x = \sqrt[5]{362+\sqrt{131043}}}$ จะได้ว่า $\displaystyle{\frac{1}{x} = \sqrt[5]{362-\sqrt{131043}}}$
ดังนั้น $\displaystyle{x^5+\frac{1}{x^5} = 724}$
ให้ $\displaystyle{a = x + \frac{1}{x}}$ จะได้ว่า
$\displaystyle{x^2+\frac{1}{x^2} = (x + \frac{1}{x})^2 - 2 = a^2 - 2}$
$\displaystyle{x^3+\frac{1}{x^3} = (x^2 + \frac{1}{x^2})(x+\frac{1}{x}) - (x+\frac{1}{x}) = a^3 - 3a}$
$\displaystyle{x^4+\frac{1}{x^4} = (x^3 + \frac{1}{x^3})(x+\frac{1}{x}) - (x^2+\frac{1}{x^2}) = a^4 -4a^2 + 2}$
$\displaystyle{x^5+\frac{1}{x^5} = (x^4 + \frac{1}{x^4})(x+\frac{1}{x}) - (x^3+\frac{1}{x^3}) = a^5 -5a^3 + 5a}$

ดังนั้นจะได้สมการพหุนาม
$a^5 - 5a^3 + 5a - 724 = 0$
$(a - 4)(a^4+4a^3+11a^2+44a+181) = 0$
เพราะฉะนั้น $a=4$ (เนื่องจาก $a>0$ เทอมหลังเป็นบวกเสมอ )

ที่เหลือก็จัดรูปครับ เหนื่อยอีกรอบ จะได้คำตอบเท่ากับ $\frac{1}{64}$
__________________
site:mathcenter.net คำค้น
ตอบพร้อมอ้างอิงข้อความนี้
  #21  
Old 21 มกราคม 2007, 05:46
sornchai sornchai ไม่อยู่ในระบบ
ลมปราณคุ้มครองร่าง
 
วันที่สมัครสมาชิก: 16 สิงหาคม 2006
ข้อความ: 290
sornchai is on a distinguished road
Post

คุณ nooonuii ครับเก่งมากเลยที่ช่วยเฉลยข้อ 10 ให้แต่ผมสงสัยว่าทำไม
($a_{ 1 }-a_{2 })+(a_{ 2 }-a_{3 })+........+(a_{n }-a_{1 })=0 $
และทำไม A = B =$ \frac{1}{2} $ ผมพื้นฐานไม่แน่นครับช่วยอธิบายหน่อย
__________________
soom soom
ตอบพร้อมอ้างอิงข้อความนี้
  #22  
Old 21 มกราคม 2007, 06:45
nooonuii nooonuii ไม่อยู่ในระบบ
ผู้พิทักษ์กฎทั่วไป
 
วันที่สมัครสมาชิก: 25 พฤษภาคม 2001
ข้อความ: 6,408
nooonuii is on a distinguished road
Post

11.
$\begin{array}{rcl} \displaystyle{ \frac{n^2 + 2n + 3}{n^4 + 6n^3+11n^2+6n} } & = & \displaystyle{\frac{n(n+1) + (n+3)}{n(n+1)(n+2)(n+3)}} \\ & = & \displaystyle{ \frac{1}{(n+2)(n+3)} + \frac{1}{n} \big( \frac{1}{(n+1)(n+2)} \big) } \\ & = & \displaystyle{\frac{1}{2} \big( \frac{1}{n} - \frac{1}{n+1} \big) - \frac{1}{2} \big( \frac{1}{n+1} - \frac{1}{n+2} \big) + \big( \frac{1}{n+2} - \frac{1}{n+3} \big) } \end{array} $

ดังนั้น

$\displaystyle{ \sum_{n=1}^{200} \frac{n^2 + 2n + 3}{n^4 + 6n^3+11n^2+6n} = \frac{1}{2} \big( 1 - \frac{1}{201} \big) - \frac{1}{2} \big( \frac{1}{2} - \frac{1}{202} \big) + \big( \frac{1}{3} - \frac{1}{203} \big) = \frac{2383625}{4121103}}$

อ้างอิง:
ข้อความเดิมของคุณ sornchai:
คุณ nooonuii ครับเก่งมากเลยที่ช่วยเฉลยข้อ 10 ให้แต่ผมสงสัยว่าทำไม
($a_{ 1 }-a_{2 })+(a_{ 2 }-a_{3 })+........+(a_{n }-a_{1 })=0 $
และทำไม A = B =$ \frac{1}{2} $ ผมพื้นฐานไม่แน่นครับช่วยอธิบายหน่อย
ลองดูที่ตัวตั้งในแต่ละวงเล็บนะครับ ผลบวกทั้งหมดจะเท่ากับ $a_1+a_2+\cdots + a_n$ ในขณะเดียวกัน ดูที่ตัวลบในแต่ละวงเล็บถ้านำมาบวกกันจะได้ $a_2 + a_3 + \cdots + a_n + a_1 = a_1 + a_2 + \cdots + a_n$ ดังนั้นพอเอามาลบกันก็เลยเป็นศูนย์ครับ

โจทย์กำหนดไว้ครับว่า $A\geq \frac{1}{2} \geq B$ แต่เราได้ว่า $A = B$ อสมการก็เลยถูกบีบให้เป็นเท่ากับครับ
__________________
site:mathcenter.net คำค้น

26 มีนาคม 2007 00:02 : ข้อความนี้ถูกแก้ไขแล้ว 1 ครั้ง, ครั้งล่าสุดโดยคุณ nongtum
ตอบพร้อมอ้างอิงข้อความนี้
  #23  
Old 21 มกราคม 2007, 12:42
หยินหยาง's Avatar
หยินหยาง หยินหยาง ไม่อยู่ในระบบ
กระบี่จักรวาล
 
วันที่สมัครสมาชิก: 06 มกราคม 2007
ข้อความ: 2,921
หยินหยาง is on a distinguished road
Post

ตอนที่ 1 ข้อ 6 ตอบข้อ ก คือ - 1
bx จะมีค่ามากกว่า 1 เสมอ เมื่อกำหนดให้ 0 a b 2b
ดังนั้น (1 - bx)/(1 + bx) จะมีค่าเป็นลบ ต่อจากนั้นนำ (1 - bx)/(1 + bx) เข้าไปในรากเพื่อไปคูณกับ (1 + ax)/(1 - ax) ซึ่งจะตัดกันหมดเหลือ
เท่ากับ 1 แต่เนื่องจากเรารู้ว่าค่าของ (1 - bx)/(1 + bx) เป็นลบเราจึงต้องคงเครื่องหมายลบไว้หน้าราก ดังนั้นคำตอบสุดท้ายจึงเป็น - 1
ตอนที่ 2 ข้อ 1 ตอบ 393
ขออภัยด้วยที่ยังไม่เฉลยวิธีทำเพราะข้าน้อยใช้ LaTeX ไม่ค่อยเป็นเวลาพิมพ์เศษส่วนมันไม่ออกเป็นเศษส่วนข้าน้อยจะพยายามลองดูและจะตามมาแสดงวิธีทำทั้งตอนที่ 1 ข้อ 6 และ ตอนที่ 2 ข้อ 1 แต่ถ้าหากใครสามารถรู้วิธีทำก็เชิญแสดงฝีมือได้เลยครับ ส่วนข้ออื่นๆ หากมีเวลาข้าน้อยจะร่วม
สัปยุทธด้วยในคราวต่อไป
ตอบพร้อมอ้างอิงข้อความนี้
  #24  
Old 21 มกราคม 2007, 13:37
passer-by passer-by ไม่อยู่ในระบบ
ผู้พิทักษ์กฎทั่วไป
 
วันที่สมัครสมาชิก: 11 เมษายน 2005
ข้อความ: 1,442
passer-by is on a distinguished road
Post

SELECTED HINTS/SOLUTIONS FOR PLANE GEOMETRY QUESTIONS

ข้อ 2 (ตอนที่ 1)
ให้ R แทน circumradius และ r แทน inradius
เราสามารถหา พื้นที่สามเหลี่ยมในโจทย์ได้ จาก (i) Herons' formula , (ii) ในเทอมของ circumradius และด้านทั้ง 3 , (iii) ในเทอมของ inradius และด้านทั้ง 3
กล่าวคือ

(i) Heron's formula : $\sqrt{s(s-a)(s-b)(s-c)} \rightarrow \frac{\sqrt{(100-c^2)(c^2-16)}}{4} \cdots(1) $

(ii) $ \text{Area}=\frac{abc}{4R} \rightarrow \frac{21c}{4R}=\frac{21c}{14r}=\frac{3c}{2r}\cdots(2) $

(iii) $ \text{Area}= rs= \frac{(10+c)r}{2}\cdots(3) $

จาก (1) =(2)=(3) เมื่อ ทำให้เหลือ c ตัวแปรเดียว จะได้ $ (c-8)(c^2-2c-20) =0 $

แต่โจทย์ต้องการ integer ดังนั้น ตอบ 8


ข้อ 17 (ตอนที่ 1)

ให้ AB,BC,CA ยาว 2006,2007,2005 หน่วย ตามลำดับ และ O เป็นจุด ศ.ก. วงกลมแนบใน

ถ้า M อยู่บน AB และ N อยู่บน BC เราจะได้ MO= MA , ON= NC (HINT : ลาก M,N ตั้งฉากกับ AC และลาก O ตั้งฉากกับ AB,BC)

จากสามเหลี่ยมคล้ายจะได้

$\frac{2006-MO}{2006}=\frac{2007-ON}{2007}=\frac{MN}{2005} $

ใช้ข้อเท็จจริงที่ว่า $ \frac{a}{b}=\frac{c}{d} \rightarrow \frac{a+c}{b+d}=\frac{a}{b}=\frac{c}{d} $
ดังนั้น

$ \frac{4013-MN}{4013}=\frac{MN}{2005} $

เมื่อ solve แล้วจะได้ MN ประมาณ 1337 หน่วย


ข้อ 28 (ตอนที่ 2) ตอบ 35 องศา ครับ แล้วเดี๋ยวว่างๆจะเอารูปข้อนี้มาแปะให้นะครับ

ข้อ 34 (ตอนที่ 2) ใช้ข้อเท็จจริงที่ว่า AB เป็นคอร์ดร่วม , PAB เป็นสามเหลี่ยมด้านเท่า และ AQB เป็นสามเหลี่ยมมุมฉาก solve หารัศมีวงกลม Q ถ้าผมคิดเลขไม่ผิด น่าจะตอบ $ 3\sqrt{2}-3 $ ครับ


ส่วนข้ออื่นๆ
ข้อ 15 ตอนที่ 1 ผมไม่ได้ตรงกับตัวเลือกเลยครับ ยังไงใครว่างช่วยเช็คให้นิดนึงนะครับ
ข้อ 19 ตอนที่ 1 ผมก็ได้ 665 วิธีเหมือนน้อง prachya ครับ
ข้อ 5 ตอนที่ 2 ผมได้ 1 ครับ โดยใช้ข้อเท็จจริงที่ว่า (-8)*X= -1/8 แล้วก็คิดจาก -7ไล่กลับไปหา -1
ข้อ 18 ตอนที่ 2 คุณ nongtum ลืมบวก 0.25 หรือเปล่าครับ
ข้อ 32 ตอนที่ 2 ผมได้ $ x^3+x $ โดย ใช้ ข้อเท็จจริงที่ว่า $ x^4+1 | x^{4k}+1 $ เมื่อ k เป็นเลขคี่
__________________
เกษียณตัวเอง ปลายมิถุนายน 2557 แต่จะกลับมาเป็นครั้งคราว
ตอบพร้อมอ้างอิงข้อความนี้
  #25  
Old 21 มกราคม 2007, 14:35
nooonuii nooonuii ไม่อยู่ในระบบ
ผู้พิทักษ์กฎทั่วไป
 
วันที่สมัครสมาชิก: 25 พฤษภาคม 2001
ข้อความ: 6,408
nooonuii is on a distinguished road
Post

ตอนที่ 2 ข้อ 13

โจทย์กำหนดให้ $a-b=3 , d-c=1,ab+cd=-2.5$
จะได้ว่า
$(a-b)^2 + (c-d)^2 = 10$
ดังนั้น $a^2+b^2+c^2+d^2 = 10 + 2(ab+cd) = 5$

ต่อไปพิจารณา
$(a+b)^2 + (c+d)^2 = a^2+b^2+c^2+d^2 +2(ab+cd) = 0$

ดังนั้น $a + b = c + d = 0$

แก้สมการเชิงเส้นได้
$a = \frac{3}{2}, b = -\frac{3}{2}, c = -\frac{1}{2},d=\frac{1}{2}$

ขอจบไว้แค่นี้นะครับเพราะมองเลขชี้กำลังไม่เห็นครับ
__________________
site:mathcenter.net คำค้น
ตอบพร้อมอ้างอิงข้อความนี้
  #26  
Old 21 มกราคม 2007, 15:08
nongtum's Avatar
nongtum nongtum ไม่อยู่ในระบบ
ผู้พิทักษ์กฎทั่วไป
 
วันที่สมัครสมาชิก: 10 เมษายน 2005
ข้อความ: 3,246
nongtum is on a distinguished road
Post

ตามแก้ตามขอบคุณฉบับแรก เฉลยหลักในหน้าแรกค่อยแก้ตอนเฉลยเสร็จและถูกทุกข้อ

คุณหยินหยาง
ขอบคุณที่ช่วยแย้งทั้งสองข้อครับ คิดทีหลังก็ตรงกันครับ
กระทู้ฝึกพิมพ์ LaTeX ไปดูได้ในห้องปัญหาการใช้เวบบอร์ดครับ มีสองสามกระทู้

คุณ passer-by

ตอนแรก
ข้อ 2 ใช้แนวคิดเดียวกันครับ
ข้อ 13 ไม่ได้คิดอ้อมๆแบบผม เยี่ยมครับ
ข้อ 15 ได้ $\frac{\cos^2\theta+1}{\cos^2\theta-1}$ ใช่ไหมครับ ถ้างั้นข้อนี้ก็ไม่มีคำตอบล่ะ
แล้วจะรอรูปข้อ 28 นะครับ
ข้อ 34 คิดได้ตรงกันครับ

ตอนที่สอง
ข้อ 5 ผมมองตรงที่ 8*x=-1/8 ออกนะ แต่ตอนแรกกลับเข้าใจผิดนึกว่ามันเปลี่ยนกลุ่มได้ ก็เลยผิดไปตามระเบียบ คำตอบคิดทีหลังตรงกันครับ
ข้อ 18 กับ 32 ผมสะเพร่าเองล่ะ เดี๋ยวค่อยแก้

คุณ nooonuii
ผมคงต้องฝึกเรื่องอสมการอีกเยอะเลยล่ะ เจอโจทย์แนวนี้ทีไรทำไม่เคยรอดสักที
ส่วนโจทย์ในไฟล์สแกนข้อ 13 ตอนที่ 2 ที่อ่านไม่ออกนั้น(ขออภัยครับ ใช้ resolution เล็กไปนิด) มันคือ $d^{8007}-a^{8001}-b^{8003}-c^{8005}$ ครับ

ยังยินดีรับคำเสนอแนะสำหรับข้ออื่นๆนะครับ

แก้ไข: แถมคำตอบให้ช่วยเช็คอีกข้อ
ตอนสอง ข้อ 33
สมมติรัศมีทรงกลมเป็น $r$ จะได้ส่วนสูงและรัศมีฐานกรวยเป็น $3r$ และ $\sqrt3r$ ตามลำดับ อัตราส่วนที่ต้องการจึงเป็น 4:9
__________________
คนไทยร่วมใจอย่าใช้ภาษาวิบัติ
ฝึกพิมพ์สัญลักษณ์สักนิด ชีวิต(คนตอบและคนถาม)จะง่ายขึ้นเยอะ (จริงๆนะ)

Stay Hungry. Stay Foolish.

21 มกราคม 2007 15:19 : ข้อความนี้ถูกแก้ไขแล้ว 1 ครั้ง, ครั้งล่าสุดโดยคุณ nongtum
ตอบพร้อมอ้างอิงข้อความนี้
  #27  
Old 21 มกราคม 2007, 15:17
nooonuii nooonuii ไม่อยู่ในระบบ
ผู้พิทักษ์กฎทั่วไป
 
วันที่สมัครสมาชิก: 25 พฤษภาคม 2001
ข้อความ: 6,408
nooonuii is on a distinguished road
Post

23. โจทย์น่าจะมีปัญหาครับ เพราะมีคำตอบเป็นจำนวนอนันต์ กรณีที่ง่ายที่สุดคือให้ตัวใดตัวหนึ่งเป็นศูนย์ จะเห็นว่าอีกสองตัวแปรที่เหลือสามารถเลือกให้เป็นจุดใดๆบนวงกลมที่มีจุดศูนย์กลางที่ $(\frac{1}{2},\frac{1}{2})$ รัศมี $\frac{\sqrt{2}}{2}$

25. จากเงื่อนไขจะได้ว่า $a+b = -1, ab = c$

ดังนั้น
$a+b = -1$
$a^2+b^2 = (a+b)^2 - 2ab = 1 - 2c$
$a^3+b^3 = (a^2+b^2)(a+b) - ab(a+b) = 3c - 1$
$a^4+b^4 = (a^3+b^3)(a+b) - ab(a^2+b^2) = 2c^2 -4c + 1$

จากเงื่อนไขโจทย์อีกอันหนึ่ง เอามากระจายแล้วจัดรูปจะได้
$$2(a^4+b^4) + 4(a^3+b^3) + ab + 2(a+b) + 4 = 0$$

แทนค่าแล้วจัดรูปอีกครั้งจะได้
$$4c^2 + 5c = 0$$

ดังนั้น $c = 0,-\frac{5}{4}$
__________________
site:mathcenter.net คำค้น

22 มกราคม 2007 00:55 : ข้อความนี้ถูกแก้ไขแล้ว 2 ครั้ง, ครั้งล่าสุดโดยคุณ nooonuii
ตอบพร้อมอ้างอิงข้อความนี้
  #28  
Old 21 มกราคม 2007, 16:10
หยินหยาง's Avatar
หยินหยาง หยินหยาง ไม่อยู่ในระบบ
กระบี่จักรวาล
 
วันที่สมัครสมาชิก: 06 มกราคม 2007
ข้อความ: 2,921
หยินหยาง is on a distinguished road
Post

ขอลุยตอนที่ 1 ก่อนนะครับ ดูเฉพาะข้อที่ผมทำได้ไม่เหมือนคนอื่นนะครับ
ข้อ 7. ได้ -( A^2+AB+B^2) ซึ่งไม่มีอยู่ในคำตอบ
ข้อ 15. ไม่มีคำตอบได้ (cos^{ 2 }q+1 ) /(cos^{ 2 }q-1 )
ข้อ 19. ไม่มีคำตอบคิดได้ 665 วิธี
ส่วนตอนทึ่ 2 ยังไม่ได้ลุย ดูแค่บางข้อ เช่น ข้อ 1 ได้คำตอบ 393 และอีกข้อ คือ ข้อ 17 ผมได้ผลบวกของรากเท่ากับ 0 ครับ คือมีรากเป็น 2 กับ -2 ไม่ตรงกับคุณ nongtum รบกวน คุณ nongtum ลองช่วยเช็คคำตอบอีกที เพราะผมใช้วิชามาร( common sense นะ) โดยดูพจน์ที่อยู่ในรากคือ
9+sqrt[80] และ 9-sqrt[80] ถ้านำมาบวกกันจะได้ = 18 ดังนั้น x ต้องเท่ากับ 2 และ 9+sqrt[80] และ 9-sqrt[80] เป็น conjugate ซึ่งกันและกัน และเมื่อ x = -2 ก็จะทำให้ผลบวก = 18 เหมือนกัน แต่ก็ไม่แน่ใจว่าจะครบทุกคำตอบหรือไม่ ถ้าผมผิดช่วยอธิบายด้วย ส่วนการแก้สมการโดยการ take log ยังไม่ได้ลองดู
ส่วนข้ออื่นๆ ขอเวลาดูหน่อยครับ แล้วจะมาลุยต่อ
ตอบพร้อมอ้างอิงข้อความนี้
  #29  
Old 21 มกราคม 2007, 16:26
nooonuii nooonuii ไม่อยู่ในระบบ
ผู้พิทักษ์กฎทั่วไป
 
วันที่สมัครสมาชิก: 25 พฤษภาคม 2001
ข้อความ: 6,408
nooonuii is on a distinguished road
Post

30. โจทย์กำหนด

$x^2+y^2 - 2x + 4y = 0 \, ...............(1)$
$x^2+y^2+xy+3y - 4 = 0 \, .........(2)$

ให้ $a = x - 1, b = y + 2$
ดังนั้น (1) เขียนได้เป็น
$a^2 + b^2 = 5 \, .........(3)$

(2) - (1) จะได้
$xy + 2x - y - 4 = 0$

จัดรูปให้อยู่ในรูปตัวแปร $a,b$ ได้เป็น

$ab = 2 \, .........(4)$

แก้ระบบสมการ (3) และ (4) ได้
$(a,b) = (-1,-2), (-2,-1), (1,2), (2,1)$

ดังนั้น
$(x,y) = (0,-4), (-1,-3), (2,0), (3,-1)$

ค่าสูงสุดของ $x^2+y^2$ จึงมีค่าเท่ากับ $16$

__________________
site:mathcenter.net คำค้น

21 มกราคม 2007 16:36 : ข้อความนี้ถูกแก้ไขแล้ว 1 ครั้ง, ครั้งล่าสุดโดยคุณ nooonuii
ตอบพร้อมอ้างอิงข้อความนี้
  #30  
Old 21 มกราคม 2007, 16:29
nongtum's Avatar
nongtum nongtum ไม่อยู่ในระบบ
ผู้พิทักษ์กฎทั่วไป
 
วันที่สมัครสมาชิก: 10 เมษายน 2005
ข้อความ: 3,246
nongtum is on a distinguished road
Icon15

ตามแก้ตามขอบคุณฉบับที่สอง เฉลยหลักในหน้าแรกค่อยแก้ตอนเฉลยเสร็จและถูกทุกข้อ

เอาเฉพาะข้อที่ผมไม่ได้พูดถึงข้างบนละกัน
ตอนแรกข้อที่ 7 ใช่ครับ มันไม่มีคำตอบอย่างที่บอก
ตอนที่สองข้อ 19 มี x=-2 อีกตัวจริงด้วย

แถมให้เช็คอีกข้อครับ
ตอนที่สอง ข้อ 30 พอแก้ระบบสมการจะได้คำตอบคือ (x,y)=(-1,-3),(0,-4),(2,0),(3,-1) ดังนั้นค่าสูงสุดที่ต้องการคือ 16 ครับ

สำหรับตอนที่สอง เหลือข้อ
15. g ที่คูณด้านหน้า เป็นอะไรครับ
28. รอรูปจากคุณ passer-by
31. ตกลง $y_1$ ที่ไม่ได้แก้คืออะไรครับ
__________________
คนไทยร่วมใจอย่าใช้ภาษาวิบัติ
ฝึกพิมพ์สัญลักษณ์สักนิด ชีวิต(คนตอบและคนถาม)จะง่ายขึ้นเยอะ (จริงๆนะ)

Stay Hungry. Stay Foolish.
ตอบพร้อมอ้างอิงข้อความนี้
ตั้งหัวข้อใหม่ Reply



กฎการส่งข้อความ
คุณ ไม่สามารถ ตั้งหัวข้อใหม่ได้
คุณ ไม่สามารถ ตอบหัวข้อได้
คุณ ไม่สามารถ แนบไฟล์และเอกสารได้
คุณ ไม่สามารถ แก้ไขข้อความของคุณเองได้

vB code is On
Smilies are On
[IMG] code is On
HTML code is Off
ทางลัดสู่ห้อง


เวลาที่แสดงทั้งหมด เป็นเวลาที่ประเทศไทย (GMT +7) ขณะนี้เป็นเวลา 21:40


Powered by vBulletin® Copyright ©2000 - 2024, Jelsoft Enterprises Ltd.
Modified by Jetsada Karnpracha